Tuesday, October 23, 2007

Putnam - 2

Well as promised here is my next Putnam problem post. Luckily I had the dilemma of choosing between two questions. Both questions came from the 1988 exam, A2 and A6. In the end I decided to post problem A6 with a solution and post problem A2 and with proof for another day.



No comments: